Quadratic Dilemma

Algebra Level 3

x 2 10 + 28 x = 7 x 5 40 x 2 \frac{x^2}{10}+\frac{28}{x}=\frac{7x}{5}-\frac{40}{x^2}

The equation above has 4 real roots x 1 x_1 , x 2 x_2 , x 3 x_3 , and x 4 x_4 such that x 1 < x 2 < x 3 < x 4 x_1<x_2<x_3<x_4 . Find the value of x 1 x 3 + x 2 x 4 x_1x_3+x_2x_4 .


The answer is -40.

This section requires Javascript.
You are seeing this because something didn't load right. We suggest you, (a) try refreshing the page, (b) enabling javascript if it is disabled on your browser and, finally, (c) loading the non-javascript version of this page . We're sorry about the hassle.

2 solutions

x 2 10 + 28 x = 7 x 5 40 x 2 Multiply both sides by 10 x 2 + 280 x = 14 x 400 x 2 Put all terms on the left. x 2 + 400 x 2 14 x + 280 x = 0 x 2 40 + 400 x 2 + 40 14 ( x 20 x ) = 0 ( x 20 x ) 2 14 ( x 20 x ) + 40 = 0 A quadratic equation of ( x 20 x ) \begin{aligned} \frac {x^2}{10} + \frac {28}x & = \frac {7x}5 - \frac {40}{x^2} & \small \color{#3D99F6} \text{Multiply both sides by }10 \\ x^2 + \frac {280}x & = 14x - \frac {400}{x^2} & \small \color{#3D99F6} \text{Put all terms on the left.} \\ x^2 + \frac {400}{x^2} - 14x + \frac {280}x & = 0 \\ {\color{#3D99F6} x^2 - 40 + \frac {400}{x^2}} + {\color{#D61F06}40} - 14 \left( x - \frac {20}x\right) & = 0 \\ {\color{#3D99F6} \left(x - \frac {20}x \right)^2} - 14 \left( x - \frac {20}x\right) + {\color{#D61F06}40} & = 0 & \small \color{#3D99F6} \text{A quadratic equation of }\left(x - \frac {20}x \right) \end{aligned}

x 20 x = 14 ± 1 4 2 4 ( 40 ) 2 = { 10 4 \begin{aligned} \implies x - \frac {20}x & = \frac {14 \pm \sqrt{14^2-4(40)}}2 = \begin{cases} 10 \\ 4 \end{cases} \end{aligned}

{ x 2 10 x 20 = 0 x = { 5 + 3 5 = x 4 5 3 5 = x 2 x 2 4 x 20 = 0 x = { 2 + 2 6 = x 3 2 2 6 = x 1 \begin{cases} x^2 - 10 x - 20 & = 0 & \implies x = \begin{cases} 5 + 3\sqrt 5 = x_4 \\ 5 - 3\sqrt 5 = x_2 \end{cases} \\ x^2 - 4 x - 20 & = 0 & \implies x = \begin{cases} 2 + 2\sqrt 6 = x_3 \\ 2 - 2\sqrt 6 = x_1 \end{cases} \end{cases}

By Vieta's formula , we have x 1 x 3 = 20 x_1x_3=-20 and x 2 x 4 = 20 x_2x_4=-20 , x 1 x 3 + x 2 x 4 = 40 \implies x_1x_3+ x_2x_4 = \boxed{-40}

Joshua Chin
Nov 4, 2017

x 2 10 + 28 x = 7 x 5 40 x 2 x 2 + 280 x = 14 x 400 x 2 x 2 + 280 x 14 x + 400 x 2 = 0 x 2 + 400 x 2 + 49 40 + 280 x 14 x = 9 ( x 20 x 7 ) 2 = 9 \begin{aligned} \frac{x^2}{10}+\frac{28}{x}&=\frac{7x}{5}-\frac{40}{x^2}\\ x^2+\frac{280}{x}&=14x-\frac{400}{x^2}\\ x^2+\frac{280}{x}-14x+\frac{400}{x^2}&=0\\ x^2+\frac{400}{x^2}+49-40+\frac{280}{x}-14x&=9\\ \left(x-\frac{20}{x}-7\right)^2&=9 \end{aligned}

So x 20 x 7 = 3 x-\frac{20}{x}-7=3\quad or x 20 x 7 = 3 \quad x-\frac{20}{x}-7=-3

Case 1:

x 20 x 7 = 3 x 2 10 x 20 = 0 x = 5 ± 3 5 \begin{aligned} x-\frac{20}{x}-7&=3\\ x^2-10x-20&=0\\ x=5\pm 3\sqrt{5} \end{aligned}

Case 2:

x 20 x 7 = 3 x 2 4 x 20 = 0 x = 2 ± 2 6 \begin{aligned} x-\frac{20}{x}-7&=-3\\ x^2-4x-20&=0\\ x=2\pm 2\sqrt{6} \end{aligned} .

When arranged in the order x 1 < x 2 < x 3 < x 4 x_1<x_2<x_3<x_4 , the roots are ( x 1 , x 2 , x 3 , x 4 ) = ( 2 2 6 , 5 3 5 , 2 + 2 6 , 5 + 3 5 ) (x_1,x_2,x_3,x_4)=(2-2\sqrt{6}, 5-3\sqrt{5}, 2+2\sqrt{6}, 5+3\sqrt{5}) .

Hence,

x 1 x 3 + x 2 x 4 = ( 2 2 6 ) ( 2 + 2 6 ) + ( 5 3 5 ) ( 5 + 3 5 ) = 20 + ( 20 ) = 40 \begin{aligned} x_1x_3+x_2x_4&=(2-2\sqrt{6})(2+2\sqrt{6})+(5-3\sqrt{5})(5+3\sqrt{5})\\ &=-20+(-20)\\ &=\boxed{-40} \end{aligned}

0 pending reports

×

Problem Loading...

Note Loading...

Set Loading...